MacLaurin/Taylor/Convergence Finals Help :(

  • MHB
  • Thread starter korth0221
  • Start date
  • Tags
    Finals
In summary: Hope that helps! :)4. Find interval of convergence of power series n=1 to infinity: (-1^(n+1)*(x-4)^n)/(n*9^n)By the ratio test, we know that a positive termed series $\displaystyle \begin{align*} \sum_{n = 0}^{\infty}{a_n} \end{align*}$ converges when $\displaystyle \begin{align*} \lim_{n \to \infty}{ \frac{a_{n+1}}{a_n} } < 1 \end{align*}$. This series is already positive termed, so we can use the ratio test directly to determine where the series
  • #1
korth0221
1
0
1. Find 2nd degree maclaurin polynomial that approximates f(x)=sec(x)

2. Find 3rd degree Taylor polynomial that approximates f(x)=(2/x) at c=1

3. Find radius of convergence of power series n=0 to infinity: ((2n)!*x^(2n))/(n!)

4. Find interval of convergence of power series n=1 to infinity: (-1^(n+1)*(x-4)^n)/(n*9^n)

My professor didn't have "time" to teach us this section so I'm very lost :( If you guys can please answer these with work that would help me a lot for this final. Thank you so much :)
 
Physics news on Phys.org
  • #2
Hello and welcome to MHB, korth0221! (Wave)

Before we get started, I do need to point out that our goal here at MHB is not to merely provide worked solutions to posted problems, but rather to engage the student to help them solve the problem(s). We also ask that no more than two question be initially posted in a thread. This helps prevent a thread from becoming convoluted and hard to follow.

So, that being said, let's look at the first question:

korth0221 said:
1. Find 2nd degree maclaurin polynomial that approximates f(x)=sec(x)

The Maclaurin series for $f$ is given by:

\(\displaystyle M(x)=\sum_{k=0}^{\infty}\left(\frac{f^{(k)}(0)}{k!}x^k\right)\)

We are asked for a partial sum...in order for $M$ to be a 2nd degree polynomial, what will our upper limit for $k$ be?
 
  • #3
Just to follow up, we want the upper limit for $k$ to be 2 in order to get a second degree polynomial...that is:

\(\displaystyle M_2(x)=\sum_{k=0}^{2}\left(\frac{f^{(k)}(0)}{k!}x^k\right)=f(0)+f'(0)x+\frac{f''(0)}{2}x^2\)

Now, we are given:

\(\displaystyle f(x)=\sec(x)\implies f(0)=1\)

And so differentiating, we get:

\(\displaystyle f'(x)=\sec(x)\tan(x)\implies f'(0)=0\)

\(\displaystyle f''(x)=\sec(x)\left(\tan^2(x)+\sec^2(x)\right)\implies f''(0)=1\)

Hence:

\(\displaystyle M_2(x)=1+\frac{1}{2}x^2\)

Here's a graph of $f$ and $M_2$ for comparison:

[DESMOS=-2,2,0.5,2]y=\sec\left(x\right);y=\frac{x^2}{2}+1[/DESMOS]

2.) For this problem, we need to use:

The Taylor series for $f$ centered at $a$ is given by:

\(\displaystyle T(x)=\sum_{k=0}^{\infty}\left(\frac{f^{(k)}(a)}{k!}(x-a)^k\right)\)

Since we are asked for a third degree polynomial, we want the partial sum:

\(\displaystyle T_3(x)=\sum_{k=0}^{3}\left(\frac{f^{(k)}(a)}{k!}(x-a)^k\right)=f(a)+f'(a)(x-a)+\frac{f''(a)}{2}(x-a)^2+\frac{f'''(a)}{6}(x-a)^3\)

We are given:

\(\displaystyle a=1\)

\(\displaystyle f(x)=\frac{2}{x}\implies f(1)=2\)

And so differentiating, we obtain:

\(\displaystyle f'(x)=-\frac{2}{x^2}\implies f'(1)=-2\)

\(\displaystyle f''(x)=\frac{4}{x^3}\implies f''(1)=4\)

\(\displaystyle f'''(x)=-\frac{12}{x^4}\implies f'''(1)=-12\)

And so we have:

\(\displaystyle T_3(x)=2-2(x-1)+2(x-1)^2-2(x-1)^3=-2x^3+8x^2-12x+8\)

Here's a a graph for comparison:

[DESMOS=0,2,0,4]y=\frac{2}{x};y=-2x^3+8x^2-12x+8[/DESMOS]

I will leave the other two problems for anyone else who is inclined to post solutions. :)
 
  • #4
korth0221 said:
2. Find 3rd degree Taylor polynomial that approximates f(x)=(2/x) at c=1

A geometric series $\displaystyle \begin{align*} \sum_{n = 0}^{\infty}{r^n} \end{align*}$ converges to $\displaystyle \begin{align*} \frac{1}{1 - r} \end{align*}$ provided that $\displaystyle \begin{align*} \left| r \right| < 1 \end{align*}$. Notice that we can write

$\displaystyle \begin{align*} f(x) &= \frac{2}{x} \\ &= 2 \left[ \frac{1}{1 - \left( 1-x \right) } \right] \end{align*}$

Now this is in the same form as the closed form of the geometric series, with $\displaystyle \begin{align*} r = 1 - x \end{align*}$ and so provided that $\displaystyle \begin{align*} \left| 1 - x \right| < 1 \end{align*}$, i.e. $\displaystyle \begin{align*} 0 < x < 2 \end{align*}$ we have

$\displaystyle \begin{align*} f( x) &= 2\left[ \frac{1}{ 1 - \left( 1 - x \right) } \right] \\ &= 2\sum_{n =0}^{\infty}{ \left( 1 - x \right) ^n } \textrm{ where } \left| 1 - x \right| < 1 \\ &= 2\sum_{n = 0}^{\infty}{ \left[ - \left( x - 1 \right) \right] ^n } \\ &= \sum_{n = 0}^{\infty}{ 2 \left( - 1 \right) ^n \left( x - 1\right) ^n } \end{align*}$
 
  • #5
korth0221 said:
3. Find radius of convergence of power series n=0 to infinity: ((2n)!*x^(2n))/(n!)

By the ratio test, we know that a positive termed series $\displaystyle \begin{align*} \sum_{n = 0}^{\infty}{a_n} \end{align*}$ converges when $\displaystyle \begin{align*} \lim_{n \to \infty}{ \frac{a_{n+1}}{a_n} } < 1 \end{align*}$.

This series might not be positive termed, as it depends what value we have for x, so we will need to use the ratio test on the absolute value series to determine where the series is absolutely convergent. So

$\displaystyle \begin{align*} \lim_{n \to \infty}{ \frac{\left| a_{n +1} \right|}{\left| a_n \right|} } &< 1 \\ \lim_{n \to \infty}{ \frac{\left| \frac{\left[ 2\left( n + 1 \right) \right] ! \, x^{2\left( n + 1 \right) }}{\left( n + 1 \right) !} \right| }{ \left| \frac{\left( 2\,n \right) ! \, x^{2\,n}}{n!} \right| } } &< 1 \\ \lim_{n \to \infty}{ \frac{\frac{\left( 2\,n + 2 \right) ! \,\left| x \right| ^{2\,n + 2}}{\left( n + 1 \right) !}}{\frac{\left( 2\,n \right) ! \,\left| x \right| ^{2\,n}}{n!}} } &< 1 \\ \lim_{n \to \infty}{ \frac{n!\,\left( 2\,n + 2 \right) ! \,\left| x \right| ^{2\,n + 2}}{\left( n + 1 \right) !\,\left( 2\,n \right) ! \,\left| x \right| ^{2\,n}} } &< 1 \\ \lim_{n \to \infty}{ \frac{n!\,\left( 2\,n + 2 \right) \left( 2\,n + 1 \right) \left( 2\,n \right) ! \,\left| x \right| ^{2\,n} \,\left| x \right| ^2}{\left( n + 1 \right) \, n! \, \left( 2\,n \right) !\,\left| x \right| ^{2\,n}} } &< 1 \\ \lim_{n \to \infty}{ \frac{\left( 2\,n + 2 \right) \left( 2\,n + 1 \right) \left| x \right| ^2}{n + 1} } &< 1 \\ \lim_{n \to \infty} 2\left( 2\,n + 1 \right) \,\left|x\right| ^2 < 1 \end{align*}$

But as $\displaystyle \begin{align*} 2\left( 2\,n + 1 \right) \to \infty \end{align*}$ as $\displaystyle \begin{align*} n \to \infty \end{align*}$, that means that there are no values of x which will make this series absolutely convergent. Thus the radius of convergence is 0.
 
  • #6
Yes, the Taylor polynomial in question 2.) can be simplified...let's begin with:

\(\displaystyle T(x)=\sum_{k=0}^{\infty}\left(\frac{f^{(k)}(a)}{k!}(x-a)^k\right)\)

Now, with \(\displaystyle f(x)=\frac{2}{x}\), we have observed that:

\(\displaystyle f'(x)=-\frac{2}{x^2}\)

\(\displaystyle f''(x)=\frac{4}{x^3}\)

\(\displaystyle f'''(x)=-\frac{12}{x^4}\)

At this point, we can state the induction hypothesis $P_n$:

\(\displaystyle f^{(n)}(x)=\frac{2(-1)^{n}n!}{x^{n+1}}\)

Having already show the base case(s) to be true, our induction step can be to differentiate both sides:

\(\displaystyle f^{(n+1)}(x)=\frac{2(-1)^{n+1}(n+1)!}{x^{(n+1)+1}}\)

We have obtained $P_{n+1}$ from $P_n$ thereby completing the proof by induction. And so we may now state:

\(\displaystyle T(x)=\sum_{k=0}^{\infty}\left(\frac{2(-1)^{k}k!}{a^{k+1}}\cdot\frac{1}{k!}(x-a)^k\right)=2\sum_{k=0}^{\infty}\left(\frac{(a-x)^k}{a^{k+1}}\right)\)
 
  • #7
korth0221 said:
4. Find interval of convergence of power series n=1 to infinity: (-1^(n+1)*(x-4)^n)/(n*9^n)

$\displaystyle \begin{align*} \sum_{n = 1}^{\infty}{ \frac{\left( -1 \right) ^{n + 1}\left( x - 4 \right) ^n}{n\,9^n} } \end{align*}$

We can say for sure that the series is absolutely convergent where $\displaystyle \begin{align*} \lim_{n \to \infty}{ \frac{\left| a_{n+1} \right| }{\left| a_n \right| } } < 1 \end{align*}$, so

$\displaystyle \begin{align*} \lim_{n \to \infty}{ \frac{\left| \frac{\left( -1 \right) ^{n + 2} \left( x - 4 \right) ^{n + 1}}{\left( n + 1 \right) \, 9^{n+1}} \right| }{ \left| \frac{\left( -1 \right) ^{n + 1}\left( x - 4 \right) ^n}{n\,9^n} \right| } } &< 1 \\ \lim_{n \to \infty}{ \frac{\frac{\left| x - 4 \right| ^{n + 1}}{\left( n + 1 \right) \, 9^{n + 1}}}{\frac{\left| x - 4 \right| ^n}{n\,9^n}} } &< 1 \\ \lim_{n \to \infty}{ \frac{n\,9^n\,\left| x - 4 \right| ^{n+1}}{\left( n + 1 \right) \, 9^{n+1}\,\left| x - 4 \right| ^n} } &< 1 \\ \lim_{n \to \infty}{ \left( \frac{n}{n + 1} \right)\,\frac{\left| x - 4 \right|}{9} } &< 1 \\ \lim_{n \to \infty}{ \left( 1 - \frac{1}{n + 1} \right) \, \frac{ \left| x - 4 \right| }{9} } &< 1 \\ \left( 1 - 0 \right) \,\frac{\left| x - 4 \right| }{9} &< 1 \\ \frac{\left| x - 4 \right| }{9} &< 1 \\ \left| x - 4 \right| &< 9 \\ -9 < x - 4 &< 9 \\ -5 < x &< 13 \end{align*}$

We know for certain that the series is absolutely convergent where $\displaystyle \begin{align*} -5 < x < 13 \end{align*}$, but the ratio test tells us nothing about the convergence at the endpoints. So we will check what happens there... $\displaystyle \begin{align*} x = -5 \end{align*}$ gives

$\displaystyle \begin{align*} \sum_{n = 1}^{\infty}{ \frac{\left( -1 \right) ^{n+1} \left( -5 - 4 \right) ^n }{ n\,9^n } } &= \sum_{n = 1}^{\infty}{ \frac{\left( - 1 \right) ^{n + 1}\left( -9 \right) ^n }{ n\,9^n } } \\ &= \sum_{n = 1}^{\infty}{ \frac{\left( -1 \right) ^{n + 1} \left( -1 \right) ^n \, 9^n}{n\,9^n} } \\ &= \sum_{n = 1}^{\infty}{ \frac{\left( -1 \right) ^{n + 2}}{n} } \\ &= \sum_{n = 1}^{\infty}{ \frac{\left( -1 \right) ^n}{n} } \end{align*}$

The Alternating Harmonic Series is well known to be convergent, as $\displaystyle \begin{align*} \frac{1}{n} \end{align*}$ is decreasing. So the series converges at $\displaystyle \begin{align*} x = -5 \end{align*}$.

When $\displaystyle \begin{align*} x = 13 \end{align*}$ we have

$\displaystyle \begin{align*} \sum_{n =1}^{\infty}{ \frac{\left( -1 \right)^{n + 1}\left( 13 - 4 \right) ^n}{n\,9^n} } &= \sum_{ n = 1}^{\infty}{ \frac{\left( -1 \right) ^{n +1} \,9^n}{n\,9^n} } \\ &= -\sum_{n = 1}^{\infty}{ \frac{\left( -1 \right) ^n}{n} } \end{align*}$

again, this has been reduced to a multiple of the Alternating Harmonic Series, so we know the series is convergent at $\displaystyle \begin{align*} x = 13 \end{align*}$.

Thus the interval of convergence is $\displaystyle \begin{align*} -5 \leq x \leq 13 \end{align*}$.
 

1. What is MacLaurin/Taylor series?

The MacLaurin/Taylor series is a mathematical representation of a function using an infinite sum of its derivatives at a specific point. It is named after mathematicians Colin Maclaurin and Brook Taylor, who developed the concept in the 18th century.

2. How do I find the coefficients of a MacLaurin/Taylor series?

To find the coefficients of a MacLaurin/Taylor series, you need to take the derivatives of the function at the given point and plug them into the formula: f(x) = f(a) + f'(a)(x-a) + f''(a)(x-a)^2/2! + f'''(a)(x-a)^3/3! + ... + f^(n)(a)(x-a)^n/n!. The coefficients will be the values of each derivative at the point a divided by the corresponding factorial.

3. What is the purpose of finding a MacLaurin/Taylor series?

The purpose of finding a MacLaurin/Taylor series is to approximate a function with a polynomial that can be easily manipulated and analyzed. This can be helpful in solving complex mathematical problems and understanding the behavior of a function around a specific point.

4. How do I determine if a MacLaurin/Taylor series converges?

A MacLaurin/Taylor series will converge if the function it represents has a finite number of derivatives at the given point and if the values of those derivatives decrease as n increases. This can be verified using tests such as the ratio test, root test, or the nth term test.

5. Can a MacLaurin/Taylor series represent any function?

No, a MacLaurin/Taylor series can only represent functions that are infinitely differentiable at the given point. It is also important to note that a MacLaurin/Taylor series is an approximation of a function and may not be an exact representation.

Similar threads

Replies
11
Views
2K
Replies
3
Views
1K
Replies
3
Views
984
Replies
5
Views
396
Replies
3
Views
1K
  • Calculus
Replies
13
Views
1K
Replies
1
Views
1K
Replies
9
Views
5K
Back
Top